Is there a dense set in $mathbb{R}$ with outer measure $1$? And what about with the Density Topology?












1












$begingroup$


I'm looking for a dense set in $mathbb{R}$ with outer measure $1$.
There is an example like that, but in $[0,1]$ Vitali set of outer-measure exactly $1$..
Also I've tried with Bernstein sets but I don't get it.



Any hint is appreciated.



Edit: I've been trying to understand the affirmation that makes F.D Tall in his articule: https://msp.org/pjm/1976/62-1/pjm-v62-n1-p25-p.pdf
In the proof of theorem 4.13, he said:




...A set of outer measure $1$ is clearly dense...




In the Density Topology, $mathbb{Q}$ is not dense. In fact, any countable subset is not dense, moreover, any nullset is closed and discrete, and viceversa. But this topology contains the euclidean topology, so if you have interesting dense subsets in $mathbb{R}$ with outer measure $1$, I would appreciate it.










share|cite|improve this question











$endgroup$












  • $begingroup$
    $(0,1)$ works. Also $[0,1]$. Also $[0,1]setminusBbb Q$.
    $endgroup$
    – Asaf Karagila
    Jan 9 at 8:16










  • $begingroup$
    But I need it to be dense in $mathbb{R}$.
    $endgroup$
    – guchihe
    Jan 9 at 8:18










  • $begingroup$
    I suspect that he's working in a measure space where the measure of the whole space is $1$ (e.g. the Cantor set with its associated Haar measure, or $[0,1]$).
    $endgroup$
    – Asaf Karagila
    Jan 9 at 8:50










  • $begingroup$
    No, the space is defined in $mathbb{R}$ and use the Lebesgue measure.
    $endgroup$
    – guchihe
    Jan 9 at 8:55










  • $begingroup$
    I also do not understand "... A set of outer measure $1$ is clearly dense..." in the paragraph following the assertions 4.13 and 4.14, because $[0,1]$ has outer measure $1$ and its complement is open in both the Euclidean and the Density topologies
    $endgroup$
    – DanielWainfleet
    Jan 10 at 0:17


















1












$begingroup$


I'm looking for a dense set in $mathbb{R}$ with outer measure $1$.
There is an example like that, but in $[0,1]$ Vitali set of outer-measure exactly $1$..
Also I've tried with Bernstein sets but I don't get it.



Any hint is appreciated.



Edit: I've been trying to understand the affirmation that makes F.D Tall in his articule: https://msp.org/pjm/1976/62-1/pjm-v62-n1-p25-p.pdf
In the proof of theorem 4.13, he said:




...A set of outer measure $1$ is clearly dense...




In the Density Topology, $mathbb{Q}$ is not dense. In fact, any countable subset is not dense, moreover, any nullset is closed and discrete, and viceversa. But this topology contains the euclidean topology, so if you have interesting dense subsets in $mathbb{R}$ with outer measure $1$, I would appreciate it.










share|cite|improve this question











$endgroup$












  • $begingroup$
    $(0,1)$ works. Also $[0,1]$. Also $[0,1]setminusBbb Q$.
    $endgroup$
    – Asaf Karagila
    Jan 9 at 8:16










  • $begingroup$
    But I need it to be dense in $mathbb{R}$.
    $endgroup$
    – guchihe
    Jan 9 at 8:18










  • $begingroup$
    I suspect that he's working in a measure space where the measure of the whole space is $1$ (e.g. the Cantor set with its associated Haar measure, or $[0,1]$).
    $endgroup$
    – Asaf Karagila
    Jan 9 at 8:50










  • $begingroup$
    No, the space is defined in $mathbb{R}$ and use the Lebesgue measure.
    $endgroup$
    – guchihe
    Jan 9 at 8:55










  • $begingroup$
    I also do not understand "... A set of outer measure $1$ is clearly dense..." in the paragraph following the assertions 4.13 and 4.14, because $[0,1]$ has outer measure $1$ and its complement is open in both the Euclidean and the Density topologies
    $endgroup$
    – DanielWainfleet
    Jan 10 at 0:17
















1












1








1





$begingroup$


I'm looking for a dense set in $mathbb{R}$ with outer measure $1$.
There is an example like that, but in $[0,1]$ Vitali set of outer-measure exactly $1$..
Also I've tried with Bernstein sets but I don't get it.



Any hint is appreciated.



Edit: I've been trying to understand the affirmation that makes F.D Tall in his articule: https://msp.org/pjm/1976/62-1/pjm-v62-n1-p25-p.pdf
In the proof of theorem 4.13, he said:




...A set of outer measure $1$ is clearly dense...




In the Density Topology, $mathbb{Q}$ is not dense. In fact, any countable subset is not dense, moreover, any nullset is closed and discrete, and viceversa. But this topology contains the euclidean topology, so if you have interesting dense subsets in $mathbb{R}$ with outer measure $1$, I would appreciate it.










share|cite|improve this question











$endgroup$




I'm looking for a dense set in $mathbb{R}$ with outer measure $1$.
There is an example like that, but in $[0,1]$ Vitali set of outer-measure exactly $1$..
Also I've tried with Bernstein sets but I don't get it.



Any hint is appreciated.



Edit: I've been trying to understand the affirmation that makes F.D Tall in his articule: https://msp.org/pjm/1976/62-1/pjm-v62-n1-p25-p.pdf
In the proof of theorem 4.13, he said:




...A set of outer measure $1$ is clearly dense...




In the Density Topology, $mathbb{Q}$ is not dense. In fact, any countable subset is not dense, moreover, any nullset is closed and discrete, and viceversa. But this topology contains the euclidean topology, so if you have interesting dense subsets in $mathbb{R}$ with outer measure $1$, I would appreciate it.







general-topology measure-theory lebesgue-measure descriptive-set-theory






share|cite|improve this question















share|cite|improve this question













share|cite|improve this question




share|cite|improve this question








edited Jan 9 at 8:59







guchihe

















asked Jan 9 at 8:13









guchiheguchihe

446




446












  • $begingroup$
    $(0,1)$ works. Also $[0,1]$. Also $[0,1]setminusBbb Q$.
    $endgroup$
    – Asaf Karagila
    Jan 9 at 8:16










  • $begingroup$
    But I need it to be dense in $mathbb{R}$.
    $endgroup$
    – guchihe
    Jan 9 at 8:18










  • $begingroup$
    I suspect that he's working in a measure space where the measure of the whole space is $1$ (e.g. the Cantor set with its associated Haar measure, or $[0,1]$).
    $endgroup$
    – Asaf Karagila
    Jan 9 at 8:50










  • $begingroup$
    No, the space is defined in $mathbb{R}$ and use the Lebesgue measure.
    $endgroup$
    – guchihe
    Jan 9 at 8:55










  • $begingroup$
    I also do not understand "... A set of outer measure $1$ is clearly dense..." in the paragraph following the assertions 4.13 and 4.14, because $[0,1]$ has outer measure $1$ and its complement is open in both the Euclidean and the Density topologies
    $endgroup$
    – DanielWainfleet
    Jan 10 at 0:17




















  • $begingroup$
    $(0,1)$ works. Also $[0,1]$. Also $[0,1]setminusBbb Q$.
    $endgroup$
    – Asaf Karagila
    Jan 9 at 8:16










  • $begingroup$
    But I need it to be dense in $mathbb{R}$.
    $endgroup$
    – guchihe
    Jan 9 at 8:18










  • $begingroup$
    I suspect that he's working in a measure space where the measure of the whole space is $1$ (e.g. the Cantor set with its associated Haar measure, or $[0,1]$).
    $endgroup$
    – Asaf Karagila
    Jan 9 at 8:50










  • $begingroup$
    No, the space is defined in $mathbb{R}$ and use the Lebesgue measure.
    $endgroup$
    – guchihe
    Jan 9 at 8:55










  • $begingroup$
    I also do not understand "... A set of outer measure $1$ is clearly dense..." in the paragraph following the assertions 4.13 and 4.14, because $[0,1]$ has outer measure $1$ and its complement is open in both the Euclidean and the Density topologies
    $endgroup$
    – DanielWainfleet
    Jan 10 at 0:17


















$begingroup$
$(0,1)$ works. Also $[0,1]$. Also $[0,1]setminusBbb Q$.
$endgroup$
– Asaf Karagila
Jan 9 at 8:16




$begingroup$
$(0,1)$ works. Also $[0,1]$. Also $[0,1]setminusBbb Q$.
$endgroup$
– Asaf Karagila
Jan 9 at 8:16












$begingroup$
But I need it to be dense in $mathbb{R}$.
$endgroup$
– guchihe
Jan 9 at 8:18




$begingroup$
But I need it to be dense in $mathbb{R}$.
$endgroup$
– guchihe
Jan 9 at 8:18












$begingroup$
I suspect that he's working in a measure space where the measure of the whole space is $1$ (e.g. the Cantor set with its associated Haar measure, or $[0,1]$).
$endgroup$
– Asaf Karagila
Jan 9 at 8:50




$begingroup$
I suspect that he's working in a measure space where the measure of the whole space is $1$ (e.g. the Cantor set with its associated Haar measure, or $[0,1]$).
$endgroup$
– Asaf Karagila
Jan 9 at 8:50












$begingroup$
No, the space is defined in $mathbb{R}$ and use the Lebesgue measure.
$endgroup$
– guchihe
Jan 9 at 8:55




$begingroup$
No, the space is defined in $mathbb{R}$ and use the Lebesgue measure.
$endgroup$
– guchihe
Jan 9 at 8:55












$begingroup$
I also do not understand "... A set of outer measure $1$ is clearly dense..." in the paragraph following the assertions 4.13 and 4.14, because $[0,1]$ has outer measure $1$ and its complement is open in both the Euclidean and the Density topologies
$endgroup$
– DanielWainfleet
Jan 10 at 0:17






$begingroup$
I also do not understand "... A set of outer measure $1$ is clearly dense..." in the paragraph following the assertions 4.13 and 4.14, because $[0,1]$ has outer measure $1$ and its complement is open in both the Euclidean and the Density topologies
$endgroup$
– DanielWainfleet
Jan 10 at 0:17












2 Answers
2






active

oldest

votes


















1












$begingroup$

In the usual (Euclidean) topology, $Bbb R$ has a dense open subset $S$ with measure $m(S)=1.$



Let $Bbb Q={q_i: iin Bbb N}.$ Let $K(1)$ be an open interval with irrational end-points, with $m(K(1))=2^{-1}$ and $q_1in K(1).$



For $nin Bbb N,$ suppose that $K(n)$ is a union of finitely many pair-wise disjoint bounded open intervals with irrational end-points, and that $m(K(n))=1-2^{-n}.$ Construct $K(n+1)$ as follows:



$K(n)$ is bounded so $Bbb Q not subset K(n).$ Let $i(n)$ be the least $iin Bbb N$ such that $q_inot in K(n).$ Since $q_{i(n)} not in overline {K(n)},$ let $U(n)$ be an open interval with irrational end-points, with $0<m(U(n))<2^{-(n+1)},$ and with $q_{i(n)}in U(n)subset Bbb R setminus K(n).$ Since $K(n)cup U(n)$ is bounded, let $ V(n)$ be an open interval with irrational end-points, with $V(n)$ disjoint from $K(n)cup U(n)$ and with $ m(V(n))=2^{-(n+1)}-m(U(n)).$



Let $K(n+1)=K(n)cup U(n)cup V(n).$



Let $S=cup_{nin Bbb N}K(n).$



It should be clear that $S$ is open and $m(S)=1.$ We show that every $q_nin S$ by induction on $n,$ as follows:



We have $q_1in S.$ For $nin Bbb N,$ if ${q_i:ile n}subset S,$ then for each $ ile n$ let $f(i)$ be the least (or any) $j$ such that $q_iin K(j),$ and let $g(n)=max {f(i):ile n}.$



Now if $q_{n+1}in K(g(n))$ then $q_{n+1}in S.$



But if $q_{n+1}not in K(g(n)),$ then, since ${q_i:ile n}subset cup_{ile n}K(f(i))=K(g(n)),$ we have, from the recursive construction of $K(g(n)+1)$ from $K(g(n)),$ that $i(g(n))=n+1,$ so $q_{n+1}in U(g(n))subset K(g(n)+1)subset S.$






share|cite|improve this answer











$endgroup$













  • $begingroup$
    By a variation on the theme of a "fat Cantor set" we can also construct a closed nowhere-dense $Tsubset Bbb R$ such that $m(T)=1,$ and $ m(Tcap (n,n+1))ne 0$ for every $nin Bbb Z.$ Although this is not the kind of set in the Q.
    $endgroup$
    – DanielWainfleet
    Jan 10 at 0:26












  • $begingroup$
    Maybe this can help me. I have to see wether $S$ is dense in the density topology or not. If it is, I just should to intersect an arbitrary dense with $S$ and i'll get a dense set with outer measure $1$ and cardinality of the dense, and I'll have the inequality... so thanks! If I get it, i'll put the answer.
    $endgroup$
    – guchihe
    Jan 10 at 2:19










  • $begingroup$
    By the Lebesgue Density Lemma, any measurable $Esubset Bbb R$ has a measurable subset $E^*$ with $m(E)=m(E^*)$ and such that, in the notation of F.D. Tall, $phi(E^*)supseteq E^*,$ which makes $E^*$ open in the Density Topology. And if $m(E)ne 0$ then $E^*$ is not empty because $m(E^*)=m(E)$.... In particular let $E=Bbb Rsetminus S.$
    $endgroup$
    – DanielWainfleet
    Jan 10 at 9:14








  • 1




    $begingroup$
    Oh it's true. In fact, just by that, I need a non-measurable set.
    $endgroup$
    – guchihe
    Jan 10 at 19:57



















7












$begingroup$

Take $mathbb{Q}cup[0,1]$, for instance.






share|cite|improve this answer









$endgroup$













  • $begingroup$
    Yes, it works. Now I need a dense more complicated. I want a dense not ''composed'' by countable denses, nor nullsets.
    $endgroup$
    – guchihe
    Jan 9 at 21:07












  • $begingroup$
    I suggest that you post it as another answer. But explain better what you're after.
    $endgroup$
    – José Carlos Santos
    Jan 9 at 21:48











Your Answer





StackExchange.ifUsing("editor", function () {
return StackExchange.using("mathjaxEditing", function () {
StackExchange.MarkdownEditor.creationCallbacks.add(function (editor, postfix) {
StackExchange.mathjaxEditing.prepareWmdForMathJax(editor, postfix, [["$", "$"], ["\\(","\\)"]]);
});
});
}, "mathjax-editing");

StackExchange.ready(function() {
var channelOptions = {
tags: "".split(" "),
id: "69"
};
initTagRenderer("".split(" "), "".split(" "), channelOptions);

StackExchange.using("externalEditor", function() {
// Have to fire editor after snippets, if snippets enabled
if (StackExchange.settings.snippets.snippetsEnabled) {
StackExchange.using("snippets", function() {
createEditor();
});
}
else {
createEditor();
}
});

function createEditor() {
StackExchange.prepareEditor({
heartbeatType: 'answer',
autoActivateHeartbeat: false,
convertImagesToLinks: true,
noModals: true,
showLowRepImageUploadWarning: true,
reputationToPostImages: 10,
bindNavPrevention: true,
postfix: "",
imageUploader: {
brandingHtml: "Powered by u003ca class="icon-imgur-white" href="https://imgur.com/"u003eu003c/au003e",
contentPolicyHtml: "User contributions licensed under u003ca href="https://creativecommons.org/licenses/by-sa/3.0/"u003ecc by-sa 3.0 with attribution requiredu003c/au003e u003ca href="https://stackoverflow.com/legal/content-policy"u003e(content policy)u003c/au003e",
allowUrls: true
},
noCode: true, onDemand: true,
discardSelector: ".discard-answer"
,immediatelyShowMarkdownHelp:true
});


}
});














draft saved

draft discarded


















StackExchange.ready(
function () {
StackExchange.openid.initPostLogin('.new-post-login', 'https%3a%2f%2fmath.stackexchange.com%2fquestions%2f3067208%2fis-there-a-dense-set-in-mathbbr-with-outer-measure-1-and-what-about-with%23new-answer', 'question_page');
}
);

Post as a guest















Required, but never shown

























2 Answers
2






active

oldest

votes








2 Answers
2






active

oldest

votes









active

oldest

votes






active

oldest

votes









1












$begingroup$

In the usual (Euclidean) topology, $Bbb R$ has a dense open subset $S$ with measure $m(S)=1.$



Let $Bbb Q={q_i: iin Bbb N}.$ Let $K(1)$ be an open interval with irrational end-points, with $m(K(1))=2^{-1}$ and $q_1in K(1).$



For $nin Bbb N,$ suppose that $K(n)$ is a union of finitely many pair-wise disjoint bounded open intervals with irrational end-points, and that $m(K(n))=1-2^{-n}.$ Construct $K(n+1)$ as follows:



$K(n)$ is bounded so $Bbb Q not subset K(n).$ Let $i(n)$ be the least $iin Bbb N$ such that $q_inot in K(n).$ Since $q_{i(n)} not in overline {K(n)},$ let $U(n)$ be an open interval with irrational end-points, with $0<m(U(n))<2^{-(n+1)},$ and with $q_{i(n)}in U(n)subset Bbb R setminus K(n).$ Since $K(n)cup U(n)$ is bounded, let $ V(n)$ be an open interval with irrational end-points, with $V(n)$ disjoint from $K(n)cup U(n)$ and with $ m(V(n))=2^{-(n+1)}-m(U(n)).$



Let $K(n+1)=K(n)cup U(n)cup V(n).$



Let $S=cup_{nin Bbb N}K(n).$



It should be clear that $S$ is open and $m(S)=1.$ We show that every $q_nin S$ by induction on $n,$ as follows:



We have $q_1in S.$ For $nin Bbb N,$ if ${q_i:ile n}subset S,$ then for each $ ile n$ let $f(i)$ be the least (or any) $j$ such that $q_iin K(j),$ and let $g(n)=max {f(i):ile n}.$



Now if $q_{n+1}in K(g(n))$ then $q_{n+1}in S.$



But if $q_{n+1}not in K(g(n)),$ then, since ${q_i:ile n}subset cup_{ile n}K(f(i))=K(g(n)),$ we have, from the recursive construction of $K(g(n)+1)$ from $K(g(n)),$ that $i(g(n))=n+1,$ so $q_{n+1}in U(g(n))subset K(g(n)+1)subset S.$






share|cite|improve this answer











$endgroup$













  • $begingroup$
    By a variation on the theme of a "fat Cantor set" we can also construct a closed nowhere-dense $Tsubset Bbb R$ such that $m(T)=1,$ and $ m(Tcap (n,n+1))ne 0$ for every $nin Bbb Z.$ Although this is not the kind of set in the Q.
    $endgroup$
    – DanielWainfleet
    Jan 10 at 0:26












  • $begingroup$
    Maybe this can help me. I have to see wether $S$ is dense in the density topology or not. If it is, I just should to intersect an arbitrary dense with $S$ and i'll get a dense set with outer measure $1$ and cardinality of the dense, and I'll have the inequality... so thanks! If I get it, i'll put the answer.
    $endgroup$
    – guchihe
    Jan 10 at 2:19










  • $begingroup$
    By the Lebesgue Density Lemma, any measurable $Esubset Bbb R$ has a measurable subset $E^*$ with $m(E)=m(E^*)$ and such that, in the notation of F.D. Tall, $phi(E^*)supseteq E^*,$ which makes $E^*$ open in the Density Topology. And if $m(E)ne 0$ then $E^*$ is not empty because $m(E^*)=m(E)$.... In particular let $E=Bbb Rsetminus S.$
    $endgroup$
    – DanielWainfleet
    Jan 10 at 9:14








  • 1




    $begingroup$
    Oh it's true. In fact, just by that, I need a non-measurable set.
    $endgroup$
    – guchihe
    Jan 10 at 19:57
















1












$begingroup$

In the usual (Euclidean) topology, $Bbb R$ has a dense open subset $S$ with measure $m(S)=1.$



Let $Bbb Q={q_i: iin Bbb N}.$ Let $K(1)$ be an open interval with irrational end-points, with $m(K(1))=2^{-1}$ and $q_1in K(1).$



For $nin Bbb N,$ suppose that $K(n)$ is a union of finitely many pair-wise disjoint bounded open intervals with irrational end-points, and that $m(K(n))=1-2^{-n}.$ Construct $K(n+1)$ as follows:



$K(n)$ is bounded so $Bbb Q not subset K(n).$ Let $i(n)$ be the least $iin Bbb N$ such that $q_inot in K(n).$ Since $q_{i(n)} not in overline {K(n)},$ let $U(n)$ be an open interval with irrational end-points, with $0<m(U(n))<2^{-(n+1)},$ and with $q_{i(n)}in U(n)subset Bbb R setminus K(n).$ Since $K(n)cup U(n)$ is bounded, let $ V(n)$ be an open interval with irrational end-points, with $V(n)$ disjoint from $K(n)cup U(n)$ and with $ m(V(n))=2^{-(n+1)}-m(U(n)).$



Let $K(n+1)=K(n)cup U(n)cup V(n).$



Let $S=cup_{nin Bbb N}K(n).$



It should be clear that $S$ is open and $m(S)=1.$ We show that every $q_nin S$ by induction on $n,$ as follows:



We have $q_1in S.$ For $nin Bbb N,$ if ${q_i:ile n}subset S,$ then for each $ ile n$ let $f(i)$ be the least (or any) $j$ such that $q_iin K(j),$ and let $g(n)=max {f(i):ile n}.$



Now if $q_{n+1}in K(g(n))$ then $q_{n+1}in S.$



But if $q_{n+1}not in K(g(n)),$ then, since ${q_i:ile n}subset cup_{ile n}K(f(i))=K(g(n)),$ we have, from the recursive construction of $K(g(n)+1)$ from $K(g(n)),$ that $i(g(n))=n+1,$ so $q_{n+1}in U(g(n))subset K(g(n)+1)subset S.$






share|cite|improve this answer











$endgroup$













  • $begingroup$
    By a variation on the theme of a "fat Cantor set" we can also construct a closed nowhere-dense $Tsubset Bbb R$ such that $m(T)=1,$ and $ m(Tcap (n,n+1))ne 0$ for every $nin Bbb Z.$ Although this is not the kind of set in the Q.
    $endgroup$
    – DanielWainfleet
    Jan 10 at 0:26












  • $begingroup$
    Maybe this can help me. I have to see wether $S$ is dense in the density topology or not. If it is, I just should to intersect an arbitrary dense with $S$ and i'll get a dense set with outer measure $1$ and cardinality of the dense, and I'll have the inequality... so thanks! If I get it, i'll put the answer.
    $endgroup$
    – guchihe
    Jan 10 at 2:19










  • $begingroup$
    By the Lebesgue Density Lemma, any measurable $Esubset Bbb R$ has a measurable subset $E^*$ with $m(E)=m(E^*)$ and such that, in the notation of F.D. Tall, $phi(E^*)supseteq E^*,$ which makes $E^*$ open in the Density Topology. And if $m(E)ne 0$ then $E^*$ is not empty because $m(E^*)=m(E)$.... In particular let $E=Bbb Rsetminus S.$
    $endgroup$
    – DanielWainfleet
    Jan 10 at 9:14








  • 1




    $begingroup$
    Oh it's true. In fact, just by that, I need a non-measurable set.
    $endgroup$
    – guchihe
    Jan 10 at 19:57














1












1








1





$begingroup$

In the usual (Euclidean) topology, $Bbb R$ has a dense open subset $S$ with measure $m(S)=1.$



Let $Bbb Q={q_i: iin Bbb N}.$ Let $K(1)$ be an open interval with irrational end-points, with $m(K(1))=2^{-1}$ and $q_1in K(1).$



For $nin Bbb N,$ suppose that $K(n)$ is a union of finitely many pair-wise disjoint bounded open intervals with irrational end-points, and that $m(K(n))=1-2^{-n}.$ Construct $K(n+1)$ as follows:



$K(n)$ is bounded so $Bbb Q not subset K(n).$ Let $i(n)$ be the least $iin Bbb N$ such that $q_inot in K(n).$ Since $q_{i(n)} not in overline {K(n)},$ let $U(n)$ be an open interval with irrational end-points, with $0<m(U(n))<2^{-(n+1)},$ and with $q_{i(n)}in U(n)subset Bbb R setminus K(n).$ Since $K(n)cup U(n)$ is bounded, let $ V(n)$ be an open interval with irrational end-points, with $V(n)$ disjoint from $K(n)cup U(n)$ and with $ m(V(n))=2^{-(n+1)}-m(U(n)).$



Let $K(n+1)=K(n)cup U(n)cup V(n).$



Let $S=cup_{nin Bbb N}K(n).$



It should be clear that $S$ is open and $m(S)=1.$ We show that every $q_nin S$ by induction on $n,$ as follows:



We have $q_1in S.$ For $nin Bbb N,$ if ${q_i:ile n}subset S,$ then for each $ ile n$ let $f(i)$ be the least (or any) $j$ such that $q_iin K(j),$ and let $g(n)=max {f(i):ile n}.$



Now if $q_{n+1}in K(g(n))$ then $q_{n+1}in S.$



But if $q_{n+1}not in K(g(n)),$ then, since ${q_i:ile n}subset cup_{ile n}K(f(i))=K(g(n)),$ we have, from the recursive construction of $K(g(n)+1)$ from $K(g(n)),$ that $i(g(n))=n+1,$ so $q_{n+1}in U(g(n))subset K(g(n)+1)subset S.$






share|cite|improve this answer











$endgroup$



In the usual (Euclidean) topology, $Bbb R$ has a dense open subset $S$ with measure $m(S)=1.$



Let $Bbb Q={q_i: iin Bbb N}.$ Let $K(1)$ be an open interval with irrational end-points, with $m(K(1))=2^{-1}$ and $q_1in K(1).$



For $nin Bbb N,$ suppose that $K(n)$ is a union of finitely many pair-wise disjoint bounded open intervals with irrational end-points, and that $m(K(n))=1-2^{-n}.$ Construct $K(n+1)$ as follows:



$K(n)$ is bounded so $Bbb Q not subset K(n).$ Let $i(n)$ be the least $iin Bbb N$ such that $q_inot in K(n).$ Since $q_{i(n)} not in overline {K(n)},$ let $U(n)$ be an open interval with irrational end-points, with $0<m(U(n))<2^{-(n+1)},$ and with $q_{i(n)}in U(n)subset Bbb R setminus K(n).$ Since $K(n)cup U(n)$ is bounded, let $ V(n)$ be an open interval with irrational end-points, with $V(n)$ disjoint from $K(n)cup U(n)$ and with $ m(V(n))=2^{-(n+1)}-m(U(n)).$



Let $K(n+1)=K(n)cup U(n)cup V(n).$



Let $S=cup_{nin Bbb N}K(n).$



It should be clear that $S$ is open and $m(S)=1.$ We show that every $q_nin S$ by induction on $n,$ as follows:



We have $q_1in S.$ For $nin Bbb N,$ if ${q_i:ile n}subset S,$ then for each $ ile n$ let $f(i)$ be the least (or any) $j$ such that $q_iin K(j),$ and let $g(n)=max {f(i):ile n}.$



Now if $q_{n+1}in K(g(n))$ then $q_{n+1}in S.$



But if $q_{n+1}not in K(g(n)),$ then, since ${q_i:ile n}subset cup_{ile n}K(f(i))=K(g(n)),$ we have, from the recursive construction of $K(g(n)+1)$ from $K(g(n)),$ that $i(g(n))=n+1,$ so $q_{n+1}in U(g(n))subset K(g(n)+1)subset S.$







share|cite|improve this answer














share|cite|improve this answer



share|cite|improve this answer








edited Jan 9 at 22:40

























answered Jan 9 at 22:20









DanielWainfleetDanielWainfleet

34.9k31648




34.9k31648












  • $begingroup$
    By a variation on the theme of a "fat Cantor set" we can also construct a closed nowhere-dense $Tsubset Bbb R$ such that $m(T)=1,$ and $ m(Tcap (n,n+1))ne 0$ for every $nin Bbb Z.$ Although this is not the kind of set in the Q.
    $endgroup$
    – DanielWainfleet
    Jan 10 at 0:26












  • $begingroup$
    Maybe this can help me. I have to see wether $S$ is dense in the density topology or not. If it is, I just should to intersect an arbitrary dense with $S$ and i'll get a dense set with outer measure $1$ and cardinality of the dense, and I'll have the inequality... so thanks! If I get it, i'll put the answer.
    $endgroup$
    – guchihe
    Jan 10 at 2:19










  • $begingroup$
    By the Lebesgue Density Lemma, any measurable $Esubset Bbb R$ has a measurable subset $E^*$ with $m(E)=m(E^*)$ and such that, in the notation of F.D. Tall, $phi(E^*)supseteq E^*,$ which makes $E^*$ open in the Density Topology. And if $m(E)ne 0$ then $E^*$ is not empty because $m(E^*)=m(E)$.... In particular let $E=Bbb Rsetminus S.$
    $endgroup$
    – DanielWainfleet
    Jan 10 at 9:14








  • 1




    $begingroup$
    Oh it's true. In fact, just by that, I need a non-measurable set.
    $endgroup$
    – guchihe
    Jan 10 at 19:57


















  • $begingroup$
    By a variation on the theme of a "fat Cantor set" we can also construct a closed nowhere-dense $Tsubset Bbb R$ such that $m(T)=1,$ and $ m(Tcap (n,n+1))ne 0$ for every $nin Bbb Z.$ Although this is not the kind of set in the Q.
    $endgroup$
    – DanielWainfleet
    Jan 10 at 0:26












  • $begingroup$
    Maybe this can help me. I have to see wether $S$ is dense in the density topology or not. If it is, I just should to intersect an arbitrary dense with $S$ and i'll get a dense set with outer measure $1$ and cardinality of the dense, and I'll have the inequality... so thanks! If I get it, i'll put the answer.
    $endgroup$
    – guchihe
    Jan 10 at 2:19










  • $begingroup$
    By the Lebesgue Density Lemma, any measurable $Esubset Bbb R$ has a measurable subset $E^*$ with $m(E)=m(E^*)$ and such that, in the notation of F.D. Tall, $phi(E^*)supseteq E^*,$ which makes $E^*$ open in the Density Topology. And if $m(E)ne 0$ then $E^*$ is not empty because $m(E^*)=m(E)$.... In particular let $E=Bbb Rsetminus S.$
    $endgroup$
    – DanielWainfleet
    Jan 10 at 9:14








  • 1




    $begingroup$
    Oh it's true. In fact, just by that, I need a non-measurable set.
    $endgroup$
    – guchihe
    Jan 10 at 19:57
















$begingroup$
By a variation on the theme of a "fat Cantor set" we can also construct a closed nowhere-dense $Tsubset Bbb R$ such that $m(T)=1,$ and $ m(Tcap (n,n+1))ne 0$ for every $nin Bbb Z.$ Although this is not the kind of set in the Q.
$endgroup$
– DanielWainfleet
Jan 10 at 0:26






$begingroup$
By a variation on the theme of a "fat Cantor set" we can also construct a closed nowhere-dense $Tsubset Bbb R$ such that $m(T)=1,$ and $ m(Tcap (n,n+1))ne 0$ for every $nin Bbb Z.$ Although this is not the kind of set in the Q.
$endgroup$
– DanielWainfleet
Jan 10 at 0:26














$begingroup$
Maybe this can help me. I have to see wether $S$ is dense in the density topology or not. If it is, I just should to intersect an arbitrary dense with $S$ and i'll get a dense set with outer measure $1$ and cardinality of the dense, and I'll have the inequality... so thanks! If I get it, i'll put the answer.
$endgroup$
– guchihe
Jan 10 at 2:19




$begingroup$
Maybe this can help me. I have to see wether $S$ is dense in the density topology or not. If it is, I just should to intersect an arbitrary dense with $S$ and i'll get a dense set with outer measure $1$ and cardinality of the dense, and I'll have the inequality... so thanks! If I get it, i'll put the answer.
$endgroup$
– guchihe
Jan 10 at 2:19












$begingroup$
By the Lebesgue Density Lemma, any measurable $Esubset Bbb R$ has a measurable subset $E^*$ with $m(E)=m(E^*)$ and such that, in the notation of F.D. Tall, $phi(E^*)supseteq E^*,$ which makes $E^*$ open in the Density Topology. And if $m(E)ne 0$ then $E^*$ is not empty because $m(E^*)=m(E)$.... In particular let $E=Bbb Rsetminus S.$
$endgroup$
– DanielWainfleet
Jan 10 at 9:14






$begingroup$
By the Lebesgue Density Lemma, any measurable $Esubset Bbb R$ has a measurable subset $E^*$ with $m(E)=m(E^*)$ and such that, in the notation of F.D. Tall, $phi(E^*)supseteq E^*,$ which makes $E^*$ open in the Density Topology. And if $m(E)ne 0$ then $E^*$ is not empty because $m(E^*)=m(E)$.... In particular let $E=Bbb Rsetminus S.$
$endgroup$
– DanielWainfleet
Jan 10 at 9:14






1




1




$begingroup$
Oh it's true. In fact, just by that, I need a non-measurable set.
$endgroup$
– guchihe
Jan 10 at 19:57




$begingroup$
Oh it's true. In fact, just by that, I need a non-measurable set.
$endgroup$
– guchihe
Jan 10 at 19:57











7












$begingroup$

Take $mathbb{Q}cup[0,1]$, for instance.






share|cite|improve this answer









$endgroup$













  • $begingroup$
    Yes, it works. Now I need a dense more complicated. I want a dense not ''composed'' by countable denses, nor nullsets.
    $endgroup$
    – guchihe
    Jan 9 at 21:07












  • $begingroup$
    I suggest that you post it as another answer. But explain better what you're after.
    $endgroup$
    – José Carlos Santos
    Jan 9 at 21:48
















7












$begingroup$

Take $mathbb{Q}cup[0,1]$, for instance.






share|cite|improve this answer









$endgroup$













  • $begingroup$
    Yes, it works. Now I need a dense more complicated. I want a dense not ''composed'' by countable denses, nor nullsets.
    $endgroup$
    – guchihe
    Jan 9 at 21:07












  • $begingroup$
    I suggest that you post it as another answer. But explain better what you're after.
    $endgroup$
    – José Carlos Santos
    Jan 9 at 21:48














7












7








7





$begingroup$

Take $mathbb{Q}cup[0,1]$, for instance.






share|cite|improve this answer









$endgroup$



Take $mathbb{Q}cup[0,1]$, for instance.







share|cite|improve this answer












share|cite|improve this answer



share|cite|improve this answer










answered Jan 9 at 8:21









José Carlos SantosJosé Carlos Santos

158k22126228




158k22126228












  • $begingroup$
    Yes, it works. Now I need a dense more complicated. I want a dense not ''composed'' by countable denses, nor nullsets.
    $endgroup$
    – guchihe
    Jan 9 at 21:07












  • $begingroup$
    I suggest that you post it as another answer. But explain better what you're after.
    $endgroup$
    – José Carlos Santos
    Jan 9 at 21:48


















  • $begingroup$
    Yes, it works. Now I need a dense more complicated. I want a dense not ''composed'' by countable denses, nor nullsets.
    $endgroup$
    – guchihe
    Jan 9 at 21:07












  • $begingroup$
    I suggest that you post it as another answer. But explain better what you're after.
    $endgroup$
    – José Carlos Santos
    Jan 9 at 21:48
















$begingroup$
Yes, it works. Now I need a dense more complicated. I want a dense not ''composed'' by countable denses, nor nullsets.
$endgroup$
– guchihe
Jan 9 at 21:07






$begingroup$
Yes, it works. Now I need a dense more complicated. I want a dense not ''composed'' by countable denses, nor nullsets.
$endgroup$
– guchihe
Jan 9 at 21:07














$begingroup$
I suggest that you post it as another answer. But explain better what you're after.
$endgroup$
– José Carlos Santos
Jan 9 at 21:48




$begingroup$
I suggest that you post it as another answer. But explain better what you're after.
$endgroup$
– José Carlos Santos
Jan 9 at 21:48


















draft saved

draft discarded




















































Thanks for contributing an answer to Mathematics Stack Exchange!


  • Please be sure to answer the question. Provide details and share your research!

But avoid



  • Asking for help, clarification, or responding to other answers.

  • Making statements based on opinion; back them up with references or personal experience.


Use MathJax to format equations. MathJax reference.


To learn more, see our tips on writing great answers.




draft saved


draft discarded














StackExchange.ready(
function () {
StackExchange.openid.initPostLogin('.new-post-login', 'https%3a%2f%2fmath.stackexchange.com%2fquestions%2f3067208%2fis-there-a-dense-set-in-mathbbr-with-outer-measure-1-and-what-about-with%23new-answer', 'question_page');
}
);

Post as a guest















Required, but never shown





















































Required, but never shown














Required, but never shown












Required, but never shown







Required, but never shown

































Required, but never shown














Required, but never shown












Required, but never shown







Required, but never shown







Popular posts from this blog

Can a sorcerer learn a 5th-level spell early by creating spell slots using the Font of Magic feature?

ts Property 'filter' does not exist on type '{}'

mat-slide-toggle shouldn't change it's state when I click cancel in confirmation window